Intégrales

Bonsoir
La première question j’ai déjà démontré avec les sommes de Riemann.
Je suis bloqué dans la deuxième question de cet exercice si vous avez une idée.
Je vous remercie.

Soit la fonction $f$ continue, strictement croissante sur l'intervalle $[ 0,a ]$ et $f( 0 )=0$. On pose $g=f^{-1}$
1- Montrer en utilisant une subdivision bien choisie du segment $[ 0,a ]$ que : $$ \int_{0}^{a}{f( t )}dt+\int_{0}^{f( a )}{g( t )}dt=af( a ).$$
2- En déduire que pour tout $\alpha \in \left[ 0,a \right]$ et $\beta \in \left[ 0,f( a ) \right]:$ $$ \alpha \beta \le \int_{0}^{\alpha }{f( t )}dt+\int_{0}^{\beta }{g( t )}dt.$$

Réponses

  • Bonjour,

    Tu fais un dessin représentant, dans un repère orthonormé, un graphe possible pour \(f\) compte-tenu des données. Tu subdivises l'intervalle \([0,a]\) pour simuler une somme de Riemann.

    Tu tournes ton dessin d'un quart de tour, tu constates qu'il y a, pour \(g\), une subdivision de \([0,f(a)]\) qui s'adapte naturellement au dessin.
  • Parles-tu de la première question ?

    je cherche à résoudre la deuxième question analytiquement et pas géométriquement.
  • Bonjour,

    Le cas $f\left( \alpha \right)\le \beta \Rightarrow \int_{0}^{f\left( \alpha \right)}{g\le \int_{0}^{\beta }{g}}$ car $g\ge 0$ et f et croissante.

    Donc facilement en trouve : $\alpha \beta \le \int_{0}^{\alpha }{f+\int_{0}^{\beta }{g}}$.

    Le cas $f\left( \alpha \right)>\beta $ je ne sais pas comment procéder.
  • Attention, il faut être un peu plus soigneux pour conclure...

    Si $f(\alpha)\leq \beta,$ on a $$\int_{0}^{\alpha}f(t)dt+\int_{0}^{\beta}g(t)dt=\int_{0}^{\alpha}f(t)dt+\int_{0}^{f(\alpha)}g(t)dt+\int_{f(\alpha)}^{\beta}g(t)dt.
    $$ Par la relation précédente et la croissance de $g,$ on a $$\int_{0}^{\alpha}f(t)dt+\int_{0}^{\beta}g(t)dt\geq \alpha f(\alpha)+ g(f(\alpha))\Big( \beta-f(\alpha) \Big)= \alpha \beta.
    $$ Si $f(\alpha)\geq \beta,$ on a $$
    \int_{0}^{\alpha}f(t)dt+\int_{0}^{\beta}g(t)dt=\int_{0}^{\alpha}f(t)dt+\int_{0}^{f(\alpha)}g(t)dt-\int_{\beta}^{f(\alpha)}g(t)dt.
    $$ Par la relation précédente et la croissance de $g,$ on a $$\int_{0}^{\alpha}f(t)dt+\int_{0}^{\beta}g(t)dt\geq \alpha f(\alpha)-g(f(\alpha))\Big( f(\alpha)-\beta \Big)= \alpha \beta.$$
  • Bravo d'avoir prouvé la question 1 avec les sommes de Riemann, c'est assez empoisonnant à formaliser.
    Si l'on suppose $f$ dérivable, c'est nettement plus facile.

    Si $q$ et $q$ sont des réels, $p>0$, $q>0$, $\frac 1p+ \frac 1q =1$, alors l'inégalité de la question 2 appliquée à $f(x)=x^{p-1}$ donne l'inégalité de Young, qui est le lemme de l'inégalité de Hölder.
    https://en.wikipedia.org/wiki/Young's_inequality_for_products
    https://en.wikipedia.org/wiki/Hölder's_inequality

    Bonne journée.
    Fr. Ch.
  • On peut toujours supposer en régularisant $f$ (compte tenu des hypothèses faites sur $f$) que $f$ est un $\mathcal{C}^{1}$ difféomorphisme qui est proche en norme infinie de $f$ sur tout compact inclus dans l'intérieur à $[0,a].$
    La première partie est alors nettement plus facile... (le résultat final s'obtient par passage à la limite).
  • La première question n'est pas si difficile à établir que vous semblez le dire :

    Si $\sigma=(a_0,\dots,a_n)$ est une subdivision quelconque de $[0,a]$ et $\sigma'=(f(a_0),\dots,f(a_n))$ son image par $f$ alors par sommation d'Abel : $$\begin{array}{rcl}\sum_{k=0}^{n-1}(a_{k+1}-a_k)f(a_k) &=& \sum_{k=0}^{n-1}(a_{k+1}f(a_{k+1})-a_kf(a_k)) - \sum_{k=0}^{n-1}(f(a_{k+1})-f(a_k))a_k \\ &=& (a_nf(a_n)-a_0f(a_0)) - \sum_{k=0}^{n-1}(f(a_{k+1})-f(a_k))g(f(a_k))\end{array}$$
    Donc $$\sum_{k=0}^{n-1}(a_{k+1}-a_k)f(a_k) + \sum_{k=0}^{n-1}\left(f(a_{k+1})-f(a_k)\right)g(f(a_k)) = af(a)$$

    Lorsque le pas de la subdivision $\sigma$ tend vers 0, celui de $\sigma'$ tend également vers 0 car $f$ est uniformément continue sur le segment $[0,a]$ d'après Heine, et donc, par le théorème des sommes de Riemann, on conclut comme voulu, puisque la somme de gauche converge vers $\int_0^a f(t)dt$ et celle de droite vers $\int_0^{f(a)}g(t)dt$.
  • Pour la deuxième question tu peux aussi fixer $\alpha$ dans $[0,a]$, et étudier la fonction $$\beta \mapsto \int_0^\alpha f(t)dt + \int_0^\beta g(t)dt- \alpha\beta$$
Connectez-vous ou Inscrivez-vous pour répondre.